Difference between revisions of "1965 AHSME Problems/Problem 3"

(Solution)
m
 
(5 intermediate revisions by 2 users not shown)
Line 7: Line 7:
 
== Solution ==
 
== Solution ==
  
We know that <math>81^{-2^{-2}}</math> is equivalent to <math>81^{\frac{1}{-2^2}}=81^{\frac{1}{4}}</math>, which is the same as <math>\sqrt[4]{81}=\boxed{\textbf{(C) }3}</math>.
+
Let us recall <math>\text{PEMDAS}</math>. We  calculate the exponent first. <math>(-2)^{-2}=\frac{1}{(-2)^2}=\frac{1}{4}</math> When we substitute, we get <math>81^{\frac{1}{4}}=\sqrt[4]{81}=\boxed{\textbf{(C) }3}</math>.
 +
 
 +
~Mathfun1000
 +
 
 +
==See Also==
 +
{{AHSME box|year=1965|num-b=2|num-a=4}}
 +
{{MAA Notice}}
 +
[[Category:AHSME]][[Category:AHSME Problems]]

Latest revision as of 21:15, 10 January 2023

Problem

The expression $(81)^{-2^{-2}}$ has the same value as:

$\textbf{(A)}\ \frac {1}{81} \qquad \textbf{(B) }\ \frac {1}{3} \qquad \textbf{(C) }\ 3 \qquad \textbf{(D) }\ 81\qquad \textbf{(E) }\ 81^4$

Solution

Let us recall $\text{PEMDAS}$. We calculate the exponent first. $(-2)^{-2}=\frac{1}{(-2)^2}=\frac{1}{4}$ When we substitute, we get $81^{\frac{1}{4}}=\sqrt[4]{81}=\boxed{\textbf{(C) }3}$.

~Mathfun1000

See Also

1965 AHSME (ProblemsAnswer KeyResources)
Preceded by
Problem 2
Followed by
Problem 4
1 2 3 4 5 6 7 8 9 10 11 12 13 14 15 16 17 18 19 20 21 22 23 24 25 26 27 28 29 30
All AHSME Problems and Solutions

The problems on this page are copyrighted by the Mathematical Association of America's American Mathematics Competitions. AMC logo.png